1
$\begingroup$

I have the following exercise:

Consider the Cauchy problem: $$\begin{cases}x' = \log x \\ x(0) = a \end{cases} \quad \text{where } a > 0 .$$ Perform a qualitative study of the solution of the problem.

Qualitative study here means the beahaviour of the maximal solution $x : (\omega_-, \omega_+) \to \mathbb R$ and its limits as $x \to \pm \infty$.

By studing the sign of $\log x$, we have that the maximal solution

  • has its graph contained in $\mathbb R \times (1, +\infty)$ and is increasing if $a > 1$
  • is constant to 1 if $a = 1$
  • has its graph contained in $\mathbb R \times (0, 1)$ and is decreasing if $a < 1$.

Essentially, the cases to be considered are only two.

Case $a > 1$. I am going to see if the maimal solution is bounded by two continuous functions. One of the is the one constant to $1$. As for the other, I observe that, being $x \ge 1$ here, we have $\log x \le x$. Now I consider the solution of the problem $$\begin{cases} u' = u \\ u(0) = a \end{cases}$$ which is $u(t) = ae^t$ and is defined over all $\mathbb R$. Then $$1 \le x(t) \le u(t) \quad \text{for every } t \in (\omega_-, \omega_+) .$$ In the current case, the maximal solution has domain $(\omega_-, \omega_+) = \mathbb R$. Both limits $\ell_- := \lim_{t \to \omega_\_} x(t)$ and $\ell_+ := \lim_{t \to \omega_+} x(t)$ do exist, being $x$ monotonic. In particular the former limit is finite; hence $$0 = \lim_{t \to \omega_\_} x'(t) = \lim_{t \to \omega_-} \log x(t) = \log \ell_- \implies \ell_- = 1 .$$ Now assume $\ell_+ < +\infty$. In that case again $$0 = \lim_{t \to \omega_+} x'(t) = \lim_{t \to \omega_+} \log x(t) = \log \ell_+ \implies \ell_+ = 1$$ which is absurd because $x(t) \ge a > 1$ for $t \ge 0$. We have to conlude that $\ell_+ = +\infty$.

Case $0 < a < 1$. For $t \le 0$, we have $a \le x(t) < 1$: that results in $\omega_- = -\infty$. The computation of $\ell_-$ is done in a similar fashion, and $\ell_- = 1$. Now I suppose $\omega_+ = +\infty$. So $$0 = \lim_{t \to \omega_+} x'(t) = \lim_{t \to \omega_+} \log x(t) = \log \ell_+ \implies \ell_+ = 1$$ which is absurd because $x(t) \le a < 1$ for $t \ge 0$. It must be $\omega_+ < +\infty$.

If my attempt is correct -- is it? --, any idea on how to compute $\ell_+ = \lim_{t \to \omega_+} x(t)$?

$\endgroup$

0

You must log in to answer this question.

Start asking to get answers

Find the answer to your question by asking.

Ask question

Explore related questions

See similar questions with these tags.